if the price of viva increases from $2.50 to $2.60 and the price of brawny is unchanged, the sales of brawny will?

Answers

Answer 1

The sales of Brawny will remain unchanged, as the amount of Brawny has not changed.

Price is one of the factors that influence the demand and sales of any product. Therefore, when the price of a product changes, the sales of that product may be impacted. However, when the price of one product changes, the sales of another product that has not changed in price may remain unaffected. In this case, since the price of Viva has increased from $2.50 to $2.60, while the amount of Brawny has remained unchanged, the sales of Brawny will likely remain unchanged.

Learn more about amount here

https://brainly.com/question/8082054

#SPJ4


Related Questions

Which can be represented by the expression 5+3(x-6)
O6 less than 3 times the sum of a number and 5
O the sum of 5 and 3 times the difference of a numbe
Othe cube of the sum of a number and 3 less than 5
the difference between 5 and the cube of the difference of a number and 6

Answers

Correct answer of the expression 5+3(x+6) is the sum of 5 and 3 times the difference of a  number by 6

what is expression?

A finite collection of symbols that are well-formed in mathematics according to context-dependent principles is called an expression or mathematical expression.

given

the expression 5+3(x+6)

it is concluded from the expression 5+3(x+6) that the sum of 5 and 3 times the difference of the number which is x by 6

so the correct answer of the expression 5+3(x+6) is the sum of 5 and 3 times the difference of a  number by 6

To know more about expression visit;-

https://brainly.com/question/14083225

#SPJ1

1/4 + 29/16 - 9735/12

Answers

Answer:

Step-by-step explanation:

To solve this problem, we need to add and subtract the fractions given. To do this, we need to find a common denominator for all of the fractions.

The least common multiple of 4, 16, and 12 is 48, so we can rewrite each fraction with a denominator of 48:

1/4 = 3/12

29/16 = 87/48

9735/12 = 4047/48

Then, we can add and subtract the fractions as follows:

3/12 + 87/48 - 4047/48 = (3 - 4047)/48 = -4044/48 = -84/16

Therefore, the final answer is -84/16, or -5 and 1/16 in simplified form.

Jocelyn and her children went into a movie theater and she bought $75.50 worth of candies and pretzels. Each candy costs $4.75 and each pretzel costs $3.50. She bought a total of 18 candies and pretzels altogether. Determine the number of candies and the number of pretzels that Jocelyn bought.

Answers

Jocelyn buys 41 candies and 25 pretzels by solving system equation using elimination method.

What is a linear equation?

The equations with one, zero, or an infinite number of solutions are known as linear equations with two variables. Each of the two variables in these equations has the largest exponent order of 1. A two-variable linear equation has the conventional form axe + by + c = 0, where x and y are the two variables. The answers can also be expressed as ordered pairs, such as (x, y).

Given that the cost of 1 candy is $4.75 and 1 pretzel is $3.50.

Jocelyn buys 18 candies and pretzels altogether with cost $75.50.

Assume that she buys x candies and y pretzels

Therefore,

x + y = 18  ......(i)

The cost of x candies and y pretzels is 4.75x + 3.50y.

4.75x + 3.50y = 75.50  .....(ii)

Solving equation (i) and (ii) by using elimination method.

Multiply equation (i) by 4.75

4.75x + 4.75y = 85.5 ....(iii)

Subtract equation (ii) from (iii)

4.75x + 4.75y = 85.5

4.75x + 3.50y = 75.50

(-)      (-)             (-)

________________

             1.25 y = 10

                    y  = 8

Putting y = 8 in equation (i)

x + 8 = 18

x = 10

To learn more about system of equations, click on below link:

brainly.com/question/14694091

#SPJ1

y=x² - 4x³
Find the value of y when x = -1.

Answers

Answer:

y = 5

Step-by-step explanation:

[tex]y=x^2-4x^3[/tex] (Given)

Plug x = -1 in the above equation, we find:

[tex]y=(-1)^2-4(-1)^3[/tex]

[tex]\rightarrow y=1-4(-1)[/tex]

[tex]\rightarrow y=1+4[/tex]

[tex]\rightarrow \red{y=5}[/tex]

Can I have some days free because I really am struggling with math these days and I have no money
Please
God loves you

Answers

I feel you, I need some free days too
God loves you too babe

Myra is taking her first hot-air balloon ride! The balloon was at an altitude of 288 meters until an air current caused it to rise another 36 meters.
What is the altitude of the balloon now?

Answers

The altitude of the balloon now is 324 meters

How to determine the altitude of the balloon now?

From the question, we have the following parameters that can be used in our computation:

Initial altitude = 288 meters

Rise = 36 meters

The altitude of the balloon now is calculated as

Current altitude = Initial altitude + Rise in altitude

Substitute the known values in the above equation, so, we have the following representation

Current altitude = 288 + 36

Evaluate the sum

Current altitude = 324

Hence, the current altitude is 324 meters

Read more about addition at

https://brainly.com/question/4721701

#SPJ1

G(x) = 4x - 5, find g(3)

Answers

To find the value of g(3), we need to substitute the value 3 into the expression for G(x) in place of x and then evaluate the resulting expression. Since G(x) = 4x - 5, we can substitute 3 for x to get G(3) = 4 * 3 - 5 = 12 - 5 = 7. Therefore, g(3) = 7.

CAN SOMEONE HELP WITH THIS QUESTION?✨

Answers

Answer:

351.5625

1,440,000

Step-by-step explanation:

[tex]\boxed{\begin{minipage}{7 cm}\underline{General form of an Exponential Function}\\\\$y=Ae^{kt}$\\\\where:\\\phantom{ww}$\bullet$ $A$ is the initial value ($y$-intercept). \\ \phantom{ww}$\bullet$ $k$ is a constant.\\ \phantom{ww}$\bullet$ $t$ is time.\\\end{minipage}}[/tex]

Given:

Doubling period = 15 minutesAt t = 120 minutes, y = 90,000

(Let t = time in minutes).

If the doubling period is 15 minutes, then at t = 135 minutes, y = 180,000:

[tex]\implies 90000=Ae^{120k}[/tex]

[tex]\implies 180000=Ae^{135k}[/tex]

Divide the second equation by the first to eliminate A, and solve for k:

[tex]\implies \dfrac{180000}{90000}=\dfrac{Ae^{135k}}{Ae^{120k}}[/tex]

[tex]\implies 2=\dfrac{e^{135k}}{e^{120k}}[/tex]

[tex]\implies 2=e^{135k} \cdot e^{-120k}[/tex]

[tex]\implies 2=e^{15k}[/tex]

[tex]\implies \ln 2 = \ln e^{15k}[/tex]

[tex]\implies \ln 2 =15k \ln e[/tex]

[tex]\implies \ln 2 =15k[/tex]

[tex]\implies k=\dfrac{1}{15}\ln 2[/tex]

Substitute t = 120, y = 90000 and the found value of k into the formula and solve for A:

[tex]\implies 90000=Ae^{\left(120 \cdot \frac{1}{15}\ln 2\right)}[/tex]

[tex]\implies 90000=Ae^{\left(8\ln 2\right)}[/tex]

[tex]\implies 90000=Ae^{\ln256}[/tex]

[tex]\implies 90000=256A[/tex]

[tex]\implies A=\dfrac{90000}{256}[/tex]

[tex]\implies A=351.5625[/tex]

Therefore, the function that models the scenario is:

[tex]\large\boxed{y=351.5625e^{\left(\frac{1}{15}t \ln 2\right)}}[/tex]

So the initial population at time t = 0 was:

351.5625

To find the size of the bacteria population after 3 hours, substitute t = 180 into the found formula:

[tex]\implies y=351.5625e^{\left(\frac{1}{15}(180) \ln 2\right)}[/tex]

[tex]\implies y=351.5625e^{\left(12 \ln 2\right)}[/tex]

[tex]\implies y=351.5625e^{\left(\ln 4096\right)}[/tex]

[tex]\implies y=351.5625 \cdot 4096[/tex]

[tex]\implies y=1440000[/tex]

Therefore, the size of the bacterial population after 3 hours was:

1,440,000

The initial population at the time t = 0 is 351.5625. And the size of the bacterial population after 3 hours is 1,440,000.

What is Exponential Growth?

An exponential function's curve is created by a pattern of data called exponential growth, which exhibits higher increases over time.

If n₀ is the initial size of a population experiencing exponential growth, then the population n(t) at time t is modeled by the function:

n(t) = n₀(e[tex])^{rt}[/tex]

Where r is the relative rate of growth expressed as a fraction of the population.

Given:

Doubling period = 15 minutes

At t = 120 minutes, n(t) = 90,000

If the doubling period is 15 minutes, then at t = 120+15 = 135 minutes,

90000 = n₀(e[tex])^{120r}[/tex]

18000 =  n₀(e[tex])^{135r}[/tex]]

To find the r:

Take ratio of both of the equations,

90000 / 18000 = n₀(e[tex])^{120r}[/tex]  /   n₀(e[tex])^{135r}[/tex]

2 = (e[tex])^{135r}[/tex] . (e[tex])^{-120r}[/tex]

r = 1/15 ln2

Substitute the value of r, t and y.

90000 = n₀(e[tex])^{120r}[/tex]

90000 = 256n₀

n₀ = 351.5652

Now, the function

n(t) = n₀(e[tex])^{rt}[/tex]

n(t) = (351.5652)(e[tex])^{(1/15)(180)(ln2)}[/tex]

n(t) = 1440000

Therefore, the initial population at the time t = 0 is 351.5625. And the size of the bacterial population after 3 hours is 1,440,000.

To learn more about the exponential growth;

https://brainly.com/question/12490064

#SPJ1

for School: Practice & Problem Solving
Amelia needs to buy some cat food. At the nearest store, 3 bags of cat food cost $6.75. How much would Amelia spend on 2 bags of cat food?

Answers

Answer:

$4.50

Step-by-step explanation:

Use a proportion:

$6.75 is to 3 bags as x is to 2 bags.

6.75/3 = x/2

3x = 2 × 6.75

x = 4.50

Answer: $4.50

Answer:

  $4.50

Step-by-step explanation:

Given 3 bags of cat food cost $6.75, you want the cost of 2 bags.

Proportion

Unless there is a volume discount (or surcharge), the price is proportional to the quantity. That means 2 bags will cost 2/3 the amount that 3 bags cost.

  cost of 2 bags = 2/3 · $6.75 = $4.50

Amelia would spend $4.50 on 2 bags of cat food.

If five standard (six-sided) dice are tossed onto the table, what is the probability that

a. all of them will show an odd number on top?

b. no aces or deuces (that means ones or twos) will show on top?

C. the five dice will show five different values on top?

If Five standard (six-sided) dice are rolled, one at a time. What is the probability that
a. the first two dice show aces, and the next three do not?
b. two of the five dice show aces, and the other three do not?

Answers

The probability of getting a specific number on the first roll of the die is 1/6. The probability of getting that same number on five successive rolls is (1/6)^5 = 1/7776. Because there are six different ways to get the same number on five rolls, multiply by 6 to get the probability of the same number on five successive rolls to be 6/7776 =1/1296.

Determining the probability that all five rolls will be different starts with asking how many ways can you choose 5 numbers from six with replacement and without repetition where order does not matter, then dividing that number by the total number of outcomes in five rolls of the die.

The total number of possible rolls is easy. There are six possible outcomes for each of the five rolls, so the total number of possible combinations after 5 rolls is 6^5 = 7776.

Now if each roll has to be a different number, consider that there are six acceptable outcomes on the first roll of the die. There are only five acceptable outcomes on the second roll, four on the third, three on the fourth, and two on the fifth. Multiplying all these together gives 6x5x4x3x2 = 6! = 720 ways to get a different number on five successive rolls of the die. This probability is 720/7776 ( 0.0926 or 9.26% or 120 times greater than getting the same number on each of five rolls.

Consider a game in which six true dice are rolled. What is the probability of obtaining exactly one ace, at least one ace and exactly two aces?

Exactly 1 ace(one) =6C1 * 5^5 = 18750 / 6^6 = 0.401877572

At Least 1 ace(one) =6C1 *5^5 +6C2 * 5^4 + 6C3 * 5^3......6C6 * 5^0= 31031/6^6 = 0.6651020233

Exactly 2 aces(ones) =6C2 * 5^4 = 9375 / 6^6 = 0.200938786

If you require two specific times(e.g. first time and second time) to be 5(probability 1/6), others to be non-5(probability 1−1/6=5/6). The probabiliy is (1/6)2(5/6)3, you have C52=10 different ways to roll two 5. Therefore the answer is 10(1/6)2(5/6)3, which is approximately 0.1608.

To learn more about five standard (six-sided) dice visit:

https://brainly.com/question/27994198?

#SPJ1

It is given that Cos(A) = 1/4 and Sin(B) = 1/2
Where A is in the 3rd quadrant and B is in the 2nd quadrant

a) Find value of Sin(A)
b) Find Value of Cos(A)
c) Find value of cos(A + B) and cos (A - B)
d) Find Value of Sin(A + B) and sin (A - B)
e) What is the quadrant of A + B and A - B
f) Find the value of Sin(2A + 2B)

Answers

The answers for the following trigonometric functions are:

a) Sin(A)=√15/4

b) Cos(A)=1/4

c) Cos(A+B)=(√3-√15)/8

Cos (A- B)=(√3+15)/8

d) Sin(A+ B) =(3√5+1)/8

Sin (A- B)==(3√5-1)/8

e) The quadrant of A+B and A-B is 4th quadrant

f)  Sin(2A + 2B)= (√15-6√5+4√3)/8

What are trigonometric functions?

The trigonometric functions in mathematics are real functions that link the angle of a right-angled triangle to the ratios of two side lengths. They are also known as circular functions, angle functions, or goniometric functions. They are widely employed in all fields of geometry-related study, including geodesy, solid mechanics, celestial mechanics, and many more. Because they are some of the most basic periodic functions, Fourier analysis is frequently employed to examine periodic events.

The sine, cosine, and tangent are the trigonometric functions that are most frequently utilized in contemporary mathematics.

Given,

Cos(A) = 1/4 and Sin(B) = 1/2

4=√1+x²

1+x²=16

x²=15

x=√15

4=1+x²

x²=3

x=√3

a) Sin(A)=√15/4

Sin(B)=1/2

b) Cos(A)=1/4

Cos(B)=√3/2

c) Cos(A+B)=CosACosB-SinASinB

=(1/4)(1/2)-(√15/4)(1/2)

=(√3-√15)/8

Cos(A-B)=CosACosB+SinASinB

=(1/4)(√3/2)+(√15/4)(1/2)

=(√3+15)/8

d) Sin(A+B)+SinACosB+CosASinB

=(√15/4)(√3/2)+(1/4)(1/2)

=(3√5+1)/8

Sin(A-B)=SinACosB-SinBCosA

=(√15/4)(√3/2)-(1/4)(1/2)

=(3√5-1)/8

e) The quadrant of A+B and A -B is 4th quadrant.

f) Sin(2A+2B)=Sin2ACos2B+Cos2ASin2B

=(√15/8)(1-√3)+((4-√15)/4)(√3/2)

=(√15-3√5+4√3-3√5)/8

=(√15-6√5+4√3)/8

To know more about trigonometric functions, visit:

https://brainly.com/question/14746686

#SPJ1


Camilla wants to attach a string of lights to the edges of her patio
for a party She does not want the string to go across the edge with
the steps. White a linear expression that represents the length of
string in feet she will need. Then find the length if x = 3. 7.EE1
4x-2
3r

Answers

The length of string in feet she will need for her patio is equal to : L{s} = 2(x + y).

What is a mathematical function, equation and expression?   function : In mathematics, a function from a set X to a set Y assigns to each element of X exactly one element of Y. The set X is called the domain of the function and the set Y is called the codomain of the function.expression : A mathematical expression is made up of terms (constants and variables) separated by mathematical operators.equation : A mathematical equation is used to equate two expressions.

Given is Camilla who wants to attach a string of lights to the edges of her patio for a party. She does not want the string to go across the edge with the steps.

Assume the shape of the patio is a rectangle with the dimensions of [x] and [y] units long. The length of string in feet she will need will be equivalent to the perimeter of the rectangle. So, the length of string in feet she will need is equal to L{s} = 2(x + y).

Therefore, the length of string in feet she will need for her patio is equal to : L{s} = 2(x + y).

To solve more questions on functions, expressions and polynomials, visit the link below -

brainly.com/question/17421223

#SPJ1

solve for x using cross multiplication x+2/4=x+5/5

Answers

Answer:

There are no solutions

Step-by-step explanation:

x + 2/4 = x + 5/5

x + 1/2 = x + 1

1/2 = 1 or 0.5 = 1

or

x + 2/4 = x + 5/5

x + 1/2 = x + 1

0 = 1/2 or 0 = 0.5

Graph the solution to the following system of inequalities.
y≤2x+3
y> -3x+5

Answers

The solution to the inequality is (0.4.3.8). The graph of the inequalities is attached with the answer.

What is inequality?

When two expressions are connected by a sign like "not equal to," "greater than," or "less than," it is said to be inequitable. The inequality shows the greater than and less than relation between variables and the numbers.

Mathematical symbols can be used to represent numbers (constants), variables, operations, functions, brackets, punctuation, and grouping. They can also denote the logical syntax's operation order and other properties.

Given inequalities are.

y≤2x+3

y> -3x+5

When we plot the graph of the inequalities on the x-y axis the two lines intersect at the point (0.4,3.8). The graph is attached with the answer below.

To know more about inequality follow

https://brainly.com/question/24372553

#SPJ1

One person from those who responded will be selected at random. Which of the following is closest to the probability that the person selected will be someone who responded no, given that the person selected is age 55 or older?
a. 0.350
b. 0.427
c. 0.462
d. 0.757
e. 0.818

Answers

Given that they are age 55 or older, it is discovered that there is a (E) 0.8181 = 81.81% probability that the person said no.

What is the probability?

Simply put, probability refers to the likelihood that something will occur.

If we don't know how an event will turn out, one can discuss the probability or likelihood of several events.

Statistics is the study of occurrences that match a probability distribution.

So, as 36 out of 44 adults aged 55 or older chose not to answer the question, the probability is given by:

p = 36/44 = 0.8181

Therefore, given that they are age 55 or older, it is discovered that there is a (E) 0.8181 = 81.81% probability that the person said no.

Know more about probability here:

https://brainly.com/question/28924396

#SPJ4

A taxi service charges $1.50 plus $0.60 per mile for a trip to the airport. The total charge is $13.50. Determine how many miles it is to the airport.

Answers

Using the linear function y = 0.6x + 1.5, he drove 20 miles to be charged $13.50

Linear Function

The parent linear function is f(x) = x, which is a line passing through the origin. In general, a linear function equation is f(x) = mx + c

where m is the slope of the equation and c is the y - intercept.

To solve this problem, we have to write an equation that models y = mx + c

c = constant = service charge = 1.50m = slope = 0.60

The linear function to model this is given as;

y = 0.6x + 1.5

Assuming a trip cost 13.50, the number of miles can be calculated as;

13.50 = 0.6x + 1.5

0.6x = 13.50 - 1.5

0.6x = 12

x = 12/0.6

x = 20

The drove 20 miles

Learn more on linear function here;

https://brainly.com/question/15602982

#SPJ1

Simon drove 55 miles per hour for 4 hours then 65 miles per hour for 3 hours how far did Simon drive in all

Answers

Answer:

415 miles

Step-by-step explanation:

Start with the speed equation:

speed = distance/time

Now solve the speed equation for distance:

distance = speed × time

Apply the speed equation solved for distance to the two parts of the trip.

4 hours at 55 mph:

distance = 55 mph × 4 hours = 220 miles

3 hours at 65 mph:

distance = 65 mph × 3 hours = 195 miles

Add the two distances to find the total distance:

total distance = 220 miles + 195 miles = 415 miles

Answer: 415 miles

Answer:

415 miles

Step-by-step explanation:

Simon drove 55 miles per hour for 4 hours then 65 miles per hour for 3 hours.

How far did he drive?

d=rt

For the first part of the trip:

d = 55 * 4 = 220 miles

For the second part of the trip:

d = 65*3 =195 miles

Add the miles together

220+195 = 415 miles

standard position intersects the unit circle at (√30/7,-√19/7). What is cot(θ)?

Answers

The cotangent of the angle is -√570/30

How to determine the cotangent of the angle?

From the question, we have the following parameters that can be used in our computation:

(√30/7,-√19/7)

This means that

(x, y) = (√30/7,-√19/7)

The cot(θ) is calculated as

cot(θ) = y/x

Substitute the known values in the above equation, so, we have the following representation

cot(θ) = (-√19/7)/(√30/7)

Evaluate

cot(θ) = -√19/√30

Rationalize

cot(θ) = -√570/30

Hence, the value of cot(θ) is -√570/30

Read more about unit circle at

https://brainly.com/question/20691579

#SPJ1

help pls will give branliiest if you explain

Answers

Answer:

slope = - 2

Step-by-step explanation:

calculate the slope m using the slope formula

m = [tex]\frac{y_{2}-y_{1} }{x_{2}-x_{1} }[/tex]

with (x₁, y₁ ) = (11, - 17) and (x₂, y₂ ) = (2, 1) ← 2 ordered pairs from the table

m = [tex]\frac{1-(-17)}{2-11}[/tex] = [tex]\frac{1+17}{-9}[/tex] = [tex]\frac{18}{-9}[/tex] = - 2

Find the GCF of each expression.
The GCF of 12y - 3 is_____.
The GFC of 4y + 10 is_____.
The GCF of 28 − 8 is _____.
The GCF of 30 + 18 is _____.

Answers

The  GCF of each expression that was given above are: 3, 2 , 4 , and 6 which can be written as:

12y-3=34y+10=228y-8=430y+18=6

What is greatest common factor?

The GCF which isthe “greatest common factor”.  can be  defined as the largest number that is a factor of two or more numbers.

Intance of this is that the GCF of 24 and 36 is 12, and this is due to the fact that the largest factor that is shared by 24 and 36 is 12.

Option 1:

The GCF of 12y-3=3 because the common factor which is the highest factor of 12 and 3 is 3

Option2

The GCF of 4y+10=2  because the common factor which is the highest factor of 4 and 10 is 2 and so on.

Option 3:

The GCF of 28 − 8= 4 because the common factor which is the highest factor of 28 and 8 is 4

Option 4:

The GCF of 30y+18=6 because the common factor which is the highest factor of 30 and 18 is 6

Learn more about GCF at:

https://brainly.com/question/219464

#SPJ1

If x)=x+7 and 9(x)=x-13.

Answers

The answer is that, save from 13, all real numbers fall into the domain of (fog)(x).

So, {x|x≠13} option 4 is correct.

What is meant by domain?

A function's domain is the collection of all potential inputs. For instance, all real numbers are in the domain of f(x)=x², and all real numbers are in the domain of g(x)=1/x, with the exception of x=0.

Let y = f(x) be a function where x and y are the independent and dependent variables. If a function f offers a means of successfully generating a single value y while using a value for x to that end, then that selected x-value is said to belong to the domain of f.

The result of multiplying the expression for f(x) by the expression for g(x) is (x + 7)(1/x-13).

This can be expressed much more clearly as (x + 7) (x – 13)

All real numbers, with the exception of 13, can be used to replace x in the expression in this condensed form. This is due to the fact that if we substitute 13 for x, our denominator will be equal to 0, making our formula illogical.

The answer is that, save from 13, all real numbers fall into the domain of (fog)(x).

So, {x|x≠13}

Therefore, option 4 is correct.

To know more about domain, visit:

https://brainly.com/question/28135761

#SPJ1

Simplify all questions
√(3-√15)²-√(3+√15)²
3√c² if c≥0
√(x²+1)²=5
-5√y² if y>0
0.5√16a² if a<0

Answers

The given equations can be simplified as :

a. -2.√15

b.  c ≥ 0

c.   x = ±2√6

d.   y < 0

e.   a > 0

How to solve inequalities ?

When solving an inequality:

you can add the same quantity to each side you can subtract the same quantity from each side you can multiply or divide each side by the same positive quantity If you multiply or divide each side by a negative quantity, the inequality symbol must be reversed.

a.

Given : √(3-√15)²-√(3+√15)²

   √(3-√15)²-√(3+√15)² = (3-√15)-(3+√15)

   √(3-√15)²-√(3+√15)² = 0 -2√15

   √(3-√15)²-√(3+√15)² = -2.√15

b .  

Given : 3√c² if c≥0

     3√c² if c≥0

    c ≥ 0

c.

Given: √(x²+1)²=5

     √(x²+1)²=5

      on Squaring both sides , we get

      (x²+1) = 25

      x²  = 24

      x = ±2√6

d.

Given : -5√y² if y>0

      -5√y² if y>0

       y < 0 [since , -5 < 0]

e.  

Given : 0.5√16a² if a<0

     0.5√16a² if a<0

      a > 0

Learn more inequalities at :

https://brainly.com/question/24372553

#SPJ1

Solve for pressure #2 using Bernoullis equation

Answers

What is the definition of Bernoulli's principle?

In fluid dynamics, Bernoulli's principle states that an increase in the speed of a fluid occurs simultaneously with a decrease in static pressure or a decrease in the fluid's potential energy.

p+12ρ v2+ρgh=konstant   Bernoulli equation(1)

Two states on a streamline are thus linked by the following equation:

p1+12ρv21+ρgh1=p2+12ρv22+ρgh2p1+12ρv21=p2+12ρv22p2=p1+12ρv21−12ρv2

Due to the constriction of the cross-section to only half the size, the flow speed is doubled. All known values can now be put in equation (5). Note that the pressure is to be used in the basic unit N/m² and the density in the unit kg/m³.

p2=p1+12ρv21−12ρv22p2=4⋅105Nm²+121000kgm³ (4ms)2–121000kgm³ (8ms)2=3.76⋅105Nm²p2=3.76 bar

To learn more about Bernoulli's principle visit:

https://brainly.com/question/24232842

#SPJ1

Solve the following
6 2/5 - 4 4/5

Answers

[tex]6\frac{2}{5} - 4\frac{4}{5}[/tex]  can be solved using subtraction of simple fraction and the final result is 8/5 .

what are simple fraction ?

A fraction in which both the numerator and the denominator consist of whole numbers.

Simplest form of a fraction:

A fraction is said to be in its simplest form if 1 is the only common factor of its numerator and denominator. For example, 8/9 ,because 1 is the only common factor of 8 and 9 in this fraction.

Simplifying proper and improper fraction

Find the highest common factor (HCF) of the numerator and  denominator.Divide both the numerator and denominator by HCF.

We simplify fractions because it is always to work or calculate when the fractions are in the simplest form.

To solve :  [tex]6\frac{2}{5} - 4\frac{4}{5}[/tex]

We know that , in simple fraction  [tex]6\frac{2}{5} - 4\frac{4}{5}[/tex] can be written as ,

[tex]6\frac{2}{5} - 4\frac{4}{5} = \frac{32}{5} - \frac{24}{5} = \frac{8}{5}[/tex]

Hence , 8/5 is the final answer .

Learn more about Simple Fractions at:

https://brainly.com/question/28336721

#SPJ1

A cactus casts a shadow 33 feet long. At the same time of day,Liam,who is 6 feet tall,casts a shadow 9 feet long,as shown. how tall is the cactus

Answers

If x = 2, y = 6, and z = 4, which expression is equivalent to 4? à 54+0-3+2=4. D Xtra 4 ... A tree is 12 feet tall and casts a shadow 9 feet long. A building nearby.

How to graph -1/2x-2/3y=3/4

Answers

Answer:

To graph an equation using the slope and y-intercept, 1) Write the equation in the form y = MX + b to find the slope m and the y-intercept (0, b). 2) Next, plot the y-intercept. 3) From the y-intercept, move up or down and left or right, depending on whether the slope is positive or negative.

Step-by-step explanation:

Part A: In two or more sentences, explain what a variable expense is and provide at least two examples.

Part B: In two or more sentences, explain what a fixed expense is and provide at least two examples.

Answers

An essential part of a business' operations is its fixed and variable expenses.

What is fixed expense?

It is an expenditure that does not fluctuate, or rather, does not alter over time.

An illustration of a fixed expense is:

1. Mortgage,

2. Rent payments,

3. Utility bills etc

Let see about Variable expense:

It is a cost that typically varies based on how the business uses the goods or services.

Examples of variable costs are as follows:

1. Sales commissions

2. Direct labor costs

3. Cost of raw materials etc

To know more about fixed expense:

https://brainly.com/question/18520563

#SPJ1

Which of the following is an equivalent expression of 14x² +18x + 5?
A 32x³ +5
B 14x²(18x + 5)
C 5(14x² + 18x)
D 18x + 14x² +5

Answers

Answer:  the correct answer is D

Step-by-step explanation:

Use the Fundamental Theorem of Calculus to find an expression for the derivative of the given function defined on the given interval, if it exists.
F(x) = â«^x t+1/t-1 dt, [1,5]

Answers

The derivative of the function is not defined overall on the interval [1, 5]

What is the Fundamental Theorem of Calculus?The essential connection between areas under curves and function derivatives is the Fundamental Theorem of Calculus. The first fundamental theorem of calculus's first section asserts that an integral of a function f over an interval with a variable upper bound can be used to derive an antiderivative or indefinite integral of f. This implies that continuous functions have antiderivatives.The second fundamental theorem of calculus, on the other hand, asserts that the integral of a function f over a specified interval equals the change of any antiderivative F between the endpoints of the interval.Here ,

       [tex]F(x)=\int_{a}^{x}\frac{t +1}{t - 1} dt\\\\F^{'}(x) = \frac{x+1}{x-1}[/tex]

The derivative of the function is not defined overall on the interval [1, 5] as x = 1 makes the derivation infinite.

To learn more about calculus, refer:

https://brainly.com/question/11237537

#SPJ4

Patel is solving 8x2 + 16x + 3 = 0. Which steps could he use to solve the quadratic equation? Select three options. (PLEASE HELP!!!)

Answers

Refer to the photo taken. You didn’t proved the options to choose from, but here’s how to solve the equation.
Other Questions
Question 10 (1 point)Under our Constitution, some powers belong to the states. What is one power of the states?Oprovide schooling and educationmake treatiescoin or print moneycreate an army 6EscribirFill in the blanks with the yo form of the appropriate verbs.decirhacerorsalirtraerver1.unos libros a la biblioteca.2. Siempre (Always)la verdad3. Por las maanas,de mi casa a las siete.4. Cuando mis amigos tienen hambre,sndwiches.5.la radio por las tardes.6. Los fines de semanapelculas con mi novia. what is the area of the polygon with explanation i need the correct present tense form of SER or ESTARmi familia... mirando una peliculaud. ... cansado In the story, Jake says it's 6:45pm but Summer says it's 6:30pm. Who is right and how do we know?Jake is right because he called the "time operator".Summer is right because she heard the clock chime.Jake is right because he asked the guard.Summer is right because she checked her watch and computer. Solve the system.x + y - 3z = -20-3y + 2z = 5z = 4 Help super confused high points for the award! HELPP PLEASEPart A. what is it the percent of change from 6,000 to 9,000? A. 36% increases B. 36% decrease C. 50% increases D. 50% decrease. Part B complete the equation given below. _% of 60 =6 Marla's mom bought a 36-pack of water bottles at the supermarket. Each bottle has 500 mL of water. How many liters of water is this? Need answer ASAP!!Find the area of the shaded region. why is global warming dangerous? Help please and thank you Find the value of XA. 40 degrees B. 50 degrees C. 60 degrees D. 100 degrees Help MOI Again PLEASE lol please help ill give brainless How did Mrs. Baker and Mrs. Bigio support Mai Thi after an eight grader was rude to her? How did Mrs. Bigio's behavior toward Mai Thi change? Which of the following words would not typically be used to describe a coup:A) ViolentB) SuddenC) RadioactiveD) Illegal Ca and F what are the name of the ionic compounds What are the five missions of the UN Security Council?PLEASE HELPWILL MARK BRAINLIEST You have 2.2 mol Xe and 2.1 mol F2, but when you carry out thereaction you end up with only 0.25 mol XeFg. What is the percentyield of this experiment?Xe(g) + 2 F2 (g) XeF, (g)